Simplify the expression (6^4)^2

Answers

Answer 1

When raising a power inside parentheses to another power, multiply the numbers:

(6^4)^2 = 6^(4x2) = 6^8

Simplified = 6^8

6^8 = 1679616

Answer 2

Answer:

[tex] \boxed{ \purple{ {6}^{8} }}[/tex]

Step-by-step explanation:

[tex] \mathsf{ ( { {6}^{4}) }^{2} }[/tex]

It is the example of Power to power law of indices.

Multiply the exponents

⇒[tex] \mathsf{ {6}^{4 \times 2} }[/tex]

Multiply the numbers

⇒[tex] \mathsf{ {6}^{8} }[/tex]

-------------------------------------------------------

[tex] \mathsf{\orange{ \underline{ power \: to \: power \: law \: of \: indices}}}[/tex]

If [tex] \mathsf{ ({x}^{a} )^{b}} [/tex] is an algebraic term then [tex] \mathsf{( {x}^{a} ) ^{b} = {x}^{a \times b} }[/tex]

i.e When an algebraic term in the index form is raised to another index , the base is raised to the power of two indices.

Hope I helped!

Best regards!!


Related Questions

Write all the factors of 32
Matj
Answer

Answers

The factors of a number are all the whole integers that can be multiplied to get that number. The factors for 32 are:

1 * 32

2 * 16

4 * 8

1, 2, 4, 8, 16, and 32

Answer: 1, 2, 4, 8, 16, and 32.

Step-by-step explanation:

Factors are what we can multiply to get the number.

Factors of 32:

1 x 32=32

2 x 16=32

4 x 8=32

Therefore, the factors of 32 are 1, 2, 4, 8, 16, and 32.

PLEASE HELP ASAP THANKS IN ADVANCE

Answers

Answer:

the answer to the question is "C"

Convert degrees to radians

Answers

Answer:

it's answer is

[tex] \frac{25}{18} [/tex]

I need help plotting this on number line

Answers

Answer:

Step-by-step explanation:

Consider the line =−−7x4y−6.
What is the slope of a line parallel to this line?

What is the slope of a line perpendicular to this line?

Answers

Answer:

4/7

Step-by-step explanation:

Original equation, in general form: -7x - 4y - 6

Rearrange to get: 4y = -7x - 6

Divide both sides by 4 to get: y = -7/4(x) - 1.5

To find the slope of a line perpendicular to another line, take the original gradient, and find the negative inverse

So for here,

Original gradient: -7/4

Negative: 7/4

Negative Inverse: 4/7 (which is our gradient)

Done!

Your friend Stacy has given you the following algebraic expression: "Subtract 20
times a number n from twice the cube of the number. What is the expression that your
friend is saying?

Answers

Answer:

Expression = 2n³ - 20n

Step-by-step explanation:

Find:

Expression

Computation:

Assume given number is 'n'

Cube of number = n³

Twice of cube = 2n³

Subtract number = 20n

Expression = 2n³ - 20n

Please Help Me!!! (WORTH 60 POINTS) Will Give Extra points out

Answers

Answer:

O C.

Step-by-step explanation:

To find a missing side length, use Pythagorean Theorem:

a^2 + b^2 = c^2 where a = 5 b = x and c = 6

5^2 + x^2 = 6^2

25 + x^2 = 36

x^2 = 36 - 25

x^2 = 11

x = 3.3

The value of x is sqrt rt 11

Answer:

√11 cm

Step-by-step explanation:

Pythagorean Thereom

a^2 + b^2= c^2

x^2 +5^2=6^2

x^2 + 25 = 36

subtract 25 from both sides

x^2=11

do the square root

x = √11

If (x - 2) and (x + 1) are factors of
x + px? + qx + 1, what is the sum of p and q?

Answers

Answer:

p + q = -3

Step-by-step explanation:

First we need to take the original equation, and factor it to a form that's easier to get two binomial factors from (i.e., let's get a quadratic):

x^3 + px^2 + qx + 1

= x (x^2 + px + q) + 1

Now that we have factored out the x, we have a quadratic trinomial which we know can be broken down into two linear binomials.  The problem gives us two linear binomials, so let's take a look.

(x - 2) (x + 1) = (x^2 + px + q)

x^2 - 2x + x -2 = x^2 + px + q

Now let's solve.

x^2 - x - 2 = x^2 + px + q

-x - 2 = px + q

From here, we can easily see that p = -1 (the coefficient of x) and q = -2.

Hence, p + q = -1 + -2 = -3.

Cheers.

Yooo I just had MAD diarrhea D:

Answers

I’m sorry I usually drink water you can lose a lot of water while doing that . Try to not a lot maybe some crackers


Find the solution of the system of equations.
2x – 10y = -28
-10x + 10y = -20
GbA

Answers

Answer:

(6, 4 )

Step-by-step explanation:

Given the 2 equations

2x - 10y = - 28 → (1)

- 10x + 10y = - 20 → (2)

Adding (1) and (2) term by term eliminates the term in y, that is

- 8x = - 48 ( divide both sides by - 8 )

x = 6

Substitute x = 6 into either of the 2 equations and evaluate for y

Substituting into (1)

2(6) - 10y = - 28

12 - 10y = - 28 ( subtract 12 from both sides )

- 10y = - 40 ( divide both sides by - 10 )

y = 4

Solution is (6, 4 )

I can’t figure this out, true or false?

Answers

Answer:

False.

Step-by-step explanation:

Because 4 is a constant .Yeah

Actually, it,s true because a constant and a coefficient are the same thing. TRUE

woman has 7 coworkers' man. How many different possible groups of four people could do the project, if one out of three is women? g

Answers

Answer: 24ways

Step-by-step explanation:

Given data:

No of men in the workplace = 7

No of women in the workplace = 1

How many ways can a group of 4 people carry out a project if on out of the 3 must be a woman.

Solution.

A group of 4 can carry out the project with one be a woman

This means there must be 3 males and 1 female in the group

= 4p3

= 24ways

The project can be carried out by 4 groups in 24 ways

7.619 by 10^-3



7.254 by 10^2​

Answers

Answer:

0.007619

0.07254

Step-by-step explanation:

1)7.619*10^-3

0.007619

2)7.254*10^2

0.07254

Explanation:

7.619*10^-3

The number here is 7.619 and the number written in scientific notation has minus 3 as its exponent.

.007.619

So the distance between the first decimal point and the second decimal is only three numbers.

Since it is exponent is minus three.

Another way to get the answer.

[tex]7.619 \times 10 {}^{ - 3} = \frac{7619}{1000} \times \frac{1}{1000} = \frac{7619}{1000000} = 0.007619 [/tex]

This applies to the second one too.

Hope this helps ;) ❤❤❤

15. Five boys went to see the CIRCUS. Four of them had Rs.5 each and the fifth boy had Re.1 more than the entrance ticket price. IF with the whole amount (which the 5 boys had), the boys were able to just buy the entrance ticket for all the 5, cost of the entrance ticket per person was​

Answers

Answer:

20+(x+1) = 5x

x=21/4

x= 5.25

The entrance ticket per person can be calculated using algebraic equation. We have create the algebraic expression as per the question.

The entrance ticket per person is Rs. 5.25.

Given:

Total boys are 5

4 boys has 5 rupee each so total rupee are [tex]=5\times 4=20[/tex].

Let the entrance ticket per boy is [tex]x[/tex].

One boy had 1 rupee more than entrance ticket [tex](x+1)[/tex].

Write the algebraic expression to calculate the entrance ticket per person.

[tex]5x=20+(x+1)\\5x=20+x+1\\5x-x=20+1\\4x=21\\x=5.25[/tex]

Thus, the entrance ticket per person is Rs. 5.25.

Learn more about algebraic expression here:

https://brainly.com/question/953809

Suppose that Y1, Y2,..., Yn denote a random sample of size n from a Poisson distribution with mean λ. Consider λˆ 1 = (Y1 + Y2)/2 and λˆ 2 = Y . Derive the efficiency of λˆ 1 relative to λˆ 2.

Answers

Answer:

The answer is "[tex]\bold{\frac{2}{n}}[/tex]".

Step-by-step explanation:

considering [tex]Y_1, Y_2,........, Y_n[/tex] signify a random Poisson distribution of the sample size of n which means is λ.

[tex]E(Y_i)= \lambda \ \ \ \ \ and \ \ \ \ \ Var(Y_i)= \lambda[/tex]

Let assume that,  

[tex]\hat \lambda_i = \frac{Y_1+Y_2}{2}[/tex]

multiply the above value by Var on both sides:

[tex]Var (\hat \lambda_1 )= Var(\frac{Y_1+Y_2}{2} )[/tex]

            [tex]=\frac{1}{4}(Var (Y_1)+Var (Y_2))\\\\=\frac{1}{4}(\lambda+\lambda)\\\\=\frac{1}{4}( 2\lambda)\\\\=\frac{\lambda}{2}\\[/tex]

now consider [tex]\hat \lambda_2[/tex] = [tex]\bar Y[/tex]

[tex]Var (\hat \lambda_2 )= Var(\bar Y )[/tex]

             [tex]=Var \{ \frac{\sum Y_i}{n}\}[/tex]

             [tex]=\frac{1}{n^2}\{\sum_{i}^{}Var(Y_i)\}\\\\=\frac{1}{n^2}\{ n \lambda \}\\\\=\frac{\lambda }{n}\\[/tex]

For calculating the efficiency divides the [tex]\hat \lambda_1 \ \ \ and \ \ \ \hat \lambda_2[/tex] value:

Formula:

[tex]\bold{Efficiency = \frac{Var(\lambda_2)}{Var(\lambda_1)}}[/tex]

                  [tex]=\frac{\frac{\lambda}{n}}{\frac{\lambda}{2}}\\\\= \frac{\lambda}{n} \times \frac {2} {\lambda}\\\\ \boxed{= \frac{2}{n}}[/tex]

If Q(x) = x2 – 2 – 2, find Q(-3).

Answers

Answer: A (10)

Step-by-step explanation:

Plug in Q(-3) into formula x^2-x-2

(-3)^2-(-3)-2= 9+3-2

=10

MY
A circle with radius of 5 cm sits inside a 11 cm x 11 cm rectangle.
Col
What is the area of the shaded region?
Round your final answer to the nearest hundredth.
MY
11 cm
Pro
Pro
Теа
5 cm
11 cm
cm2
2 of 4 OOO


Help

Answers

Step-by-step explanation:

Hi, there!!!

According to the question we must find the area of shaded region, but we must find area of circle and rectangle to find area of shaded region,

So, let's simply work with it,

Firstly, finding the area of rectangle,

length = 11cm.

breadth = 11cm.

now, area= length× breadth.

or, a = 11cm× 11cm.

a= 121cm^2

Now, let's work out the area of circle.

radius= 5cm

and pi. = 3.14 {using pi value as 3.14}

now,

area of a circle = pi× r^2

or, a= 3.14×5^2

or, a = 78.5 cm^2.

Therefore, The area of a circle is 78.5cm^2.

Now lastly finding the area of shadedregion,

area of shaded region = area of rectangle - area of circle.

or, area of shaded region = 121cm^2 - 78.5cm^2

Therefore, the area of shaded region is 42.5 cm^2.

Hope it helps...

Suppose that the manufacturer of a gas clothes dryer has found that, when the unit price is p dollars, the revenue R (in dollars) is R(P) = - 8p^2 + 24,000p. What unit
price should be established for the dryer to maximize revenue? What is the maximum revenue?
The unit price that should be established to maximize revenue is $|
(Simplify your answer.)

Answers

Here we have a problem of maximization and quadratic equations.

The unit prize that maximizes the revenue is $1,500, and the maximum revenue is $18,000.

We know that the revenue equation is:

R(P) = - 8p^2 + 24,000p

Where the variable p is the price.

Now we want to find the value of p that maximizes the revenue.

To do it, we can see that the revenue equation is a quadratic equation with a negative leading coefficient.

This means that the arms of the graph will go downwards, then the maximum point of the graph will be at the vertex.

Remember that for an equation like:

y = a*x^2 + b*x+ c

The x-value of the vertex is at:

x = -b/(2*a)

Then for the equation:

R(P) = - 8p^2 + 24,000p

The vertex is at:

p = -(24,000)/(2*-8) = 1,500

The value of p that maximizes the revenue is p = $1,500

To get the maximum revenue, we need to evaluate the revenue equation in that p value.

R(1,500) =  - 8*(1,500)^2 + 24,000*1,500 = 18,000

And the revenue equation is in dollars, then the maximum revenue is 18,000 dollars.

If you want to learn more, you can read:

https://brainly.com/question/18269297

p = 1500  $    the unit price

R(p) = 18000000 $ maximum revenue

We will use two different procedures to calculate the maximum revenue.

That is equivalent to solve the problem and after that to test the solution

The first one is:

R(p) = - 8*p² + 24000*p

we realize that R(p) is a quadratic function ( a parabola) of the form:

y = a*x² + b*x + c    ( c = 0 in this case)

We also know that  as the coefficient of p² is negative the parabola opens downwards then the vertex is a maximum value for R(p), and the x coordinate of p is:

x = p =  - b/2*a   then by substitution

p = - ( 24000)/ 2 ( - 8)

p = 1500  $   and for that value of p

R(p) = - 8 ( 1500)² + 24000* (1500)  = - 18000000 + 36000000

R(p) = 18000000 $

The second procedure is solving with the help of derivatives.

R(p) =  - 8*p² + 24000*p

Tacking derivatives on both sides of the equation we get:

R´(p) = -16p  +  24000

If R´(p) = 0    then    -16p  +  24000 = 0

p = 24000/ 16         p  =  1500

if we check for the second derivative

R´´(p) = -16          -16 < 0  therefore there is a maximum value for R(p) when p = 1500, and that value is:

By substitution in R(p)

R(p) = -8 *(1500)² + 24000* 1500

R(p) =  - 18000000 + 36000000

R(p) = 18000000 $

3x7-12-2=7?


I have no clue please help...

Answers

Answer:

X = 28/3, or 9 1/3 or 9.3

Step-by-step explanation:

Answer:

Step-by-step explanation:

4. Create your own scenario for the variable expression below. Then, suggest values for the variables and solve. 14x + 12y​

Answers

Answer:

Cost of pencil = $20

Cost of copy = $6

Step-by-step explanation:

Statement.

Gill buys 14 copy and 12 pencils and pays a total $324, if the value of 1 copy and 1 pencil is $26, find cost of copy and pencil.

Computation:

Assume.

Cost of copy = x

Cost of pencil = y

So,

x + y = 26.......Eq1

And

14x + 12y​ = 324.........Eq2

From Eq1 ad Eq2

Cost of pencil = $20

So,

Cost of copy = $6

Niall and Zayn buy 14 concert tickets for them and their friends to go see 5sos and 12 concert tickets for them and their friends to go see Little Mix with a total cost of $648. If the value of 1 5sos ticket and 1 Little Mix ticket is $52, and the Little Mix ticket is $4 more than the 5sos ticket, find cost of both tickets.

5sos = x

Little Mix = y

52 / 2 = 26

26 - 2 = 24

26 + 2 = 26

x = 24

y = 28

5sos tickets = $24 each

Little Mix tickets = $26 each

A team wishes to purchase 10 shirts of the same color. A store sells shirts in 3 different colors. What must the inventory of the store be in order to conclude that there are at least 10 shirts in one of the three colors?

Answers

Answer:

30

Step-by-step explanation:

HELP
PLSFind all the missing elements:

Answers

Answer:

a = 6.7 , c = 2.0

Step-by-step explanation:

For side a

To find the missing side a we use the sine rule

[tex] \frac{ |b| }{ \sin(B) } = \frac{ |a| }{ \sin(A) } [/tex]

From the question

B = 58°

b = 6

A = 109°

Substituting the values into the above formula we have

[tex] \frac{6}{ \sin(58) } = \frac{ |a| }{ \sin(109) } [/tex]

[tex] |a| \sin(58) = 6\sin(109) [/tex]

Divide both sides by sin 58°

[tex] |a| = \frac{6 \sin(108) }{ \sin(58) } [/tex]

a = 6.728791

a = 6.7 to the nearest tenth

For side c

To find side c we use the sine rule

That's

[tex] \frac{ |b| }{ \sin(B) } = \frac{ |c| }{ \sin(C) } [/tex]

C = 13°

[tex] \frac{6}{ \sin(58) } = \frac{ |c| }{ \sin(13) } [/tex]

[tex] |c| \sin(58) = 6 \sin(13) [/tex]

Divide both sides by sin 58°

[tex] |c| = \frac{6 \sin(13) }{ \sin(58) } [/tex]

c = 1.591544

c = 2.0 to the nearest tenth

Hope this helps you

Answer:

B=58 a=6.7 c=1.6

Step-by-step explanation:

It was right on Acellus

Sorry I cant give a better explanation but this unit is killing me.

Intro to Translations
Acellus
Find the image of the given point
under the given translation.
P(-1,2)
T(x, y) = (x + 2, y - 4)
P' = ([?], [])
Enter the number that belongs
in the green box.

Answers

Answer:

(1,-2)

Step-by-step explanation:

P(-1,2) and (x, y) -> (x + 2, y - 4). Plugging in x and y in the transformation, the transformed points are (-1+2, 2-4) = (1,-2)

3. The length of a rectangle is 4 inches more than its width. The area of the rectangle is equal to 5 inches more than 2 times the perimeter. Find the length and width of the rectangle.

Answers

Answer:
Length: sqrt13 + 4
Width: sqrt13
Step-by-step explanation:
Denote the width as x, then the length is x+4. The area of the rectangle will be x(x+4), and the perimeter will then be 4x +8. Hence, you can write the equation x(x+4) - 5 = 4x+8. Solving, you get x = sqrt13.

Lesson 9.6: Steady-State Analysis.) Consider a particular data set of 100,000 stationary waiting times obtained from a large queueing system. Suppose your goal is to get a confidence interval for the unknown mean. Would you rather use (a) 50 batches of 2000 observations or (b) 10000 batches of 10 observations each?

Answers

Answer:

I would rather use:

(b) 10,000 batches of 10 observations each.

Step-by-step explanation:

It is easier to have 10,000 batches of 10 observations each than to have 50 batches of 2,000 observations.  Human errors are reduced with fewer observations. For example, Hadoop, a framework used for storing and processing big data, relies on batch processing.  Using batch processing that divides the 100,000 stationary waiting times into 10 observations with 10,000 batches each is more efficient than having 2,000 observations with 50 batches each.

Solving Equations by Dividing 2) 9x= -135 Solve for x. 0 -144 O 126 O 15 0 -15

Answers

X = 15
Explanation:
9x = -135
Divide by 9 on both sides
X = -15
It is -15 since 135 is negative

[tex]\huge\text{Hey there!}[/tex]

[tex]\huge\boxed{\mathsf{9x = -135}}[/tex]

[tex]\huge\boxed{\text{DIVIDE 9 to BOTH SIDES}}[/tex]

[tex]\huge\boxed{\mathsf{\dfrac{9x}{9}= \dfrac{-135}{9}}}[/tex]

[tex]\huge\boxed{\mathsf{\bullet \ CANCEL: \dfrac{9}{9}\ because\ it \ gives\ you\ 1}}[/tex]

[tex]\huge\boxed{\bullet\ \mathsf{KEEP: \dfrac{-135}{9}\ because\ it\ helps\ solve \ for}}\\\huge\boxed{\mathsf{the\ x-value}}[/tex]

[tex]\huge\boxed{\mathsf{x = \dfrac{-135}{9}}}\\\\\huge\boxed{\mathsf{\dfrac{-135}{9}= x}}}[/tex]

[tex]\huge\boxed{Simplify \ it\uparrow}[/tex]

[tex]\huge\boxed{\mathsf{x = \bf -15}}[/tex]

[tex]\huge\boxed{\textsf{Therefore, your answer is: Option D. -15 }}\huge\checkmark[/tex]

[tex]\huge\text{Good luck on your assignment \& enjoy your day!}[/tex]

~[tex]\huge\boxed{\frak{Amphitrite1040:)}}[/tex]

Please help with questions 1-4.

Answers

Observe the quantities, ie. see what happens if you increase one or decrease the other.

To simplify imagination, expose one quantity. I will expose y.

1.

[tex]-\frac{y}{4}=2x\implies y=-8x[/tex]

So what is their variation or proportion? If I increase x then y is getting more and more negative. So is this direct or inverse or nothing? It is direct.

2.

[tex]14x=\frac{14}{y}\implies y=x, y\neq0[/tex] again if I increase x then y will match, will also increase. This is again direct.

3.

[tex]y=\frac{13}{x}, x\neq0[/tex] this time if I increase x, y will get smaller. When x is exactly 13, y will be 1 and when x is 10000, y will be 0.0013. This is inverse. One quantity gets really small when other quantity gets really big.

4.

[tex]y=x-2[/tex] if I increase x then y will also increase even though by slightly less (-2) it will still increase. However since there is no multiplication this is not a direct variation nor is it inverse. It is nothing/no-variation.

Hope this helps :)


1 What is the product of 2r^2- 5 and 3r

Answers

Answer:

Step-by-step explanation:

3r(2r^2 - 5)

6r^(2 + 1) - 5*3r

6r^3 - 15r

Please give me the answer ASAP The average of 5 numbers is 7. If one of the five numbers is removed, the average of the four remaining numbers is 6. What is the value of the number that was removed Show Your Work

Answers

Answer:

The removed number is 11.

Step-by-step explanation:

Given that the average of 5 numbers is 7. So you have to find the total values of 5 numbers :

[tex]let \: x = total \: values[/tex]

[tex] \frac{x}{5} = 7[/tex]

[tex]x = 7 \times 5[/tex]

[tex]x = 35[/tex]

Assuming that the total values of 5 numbers is 35. Next, we have to find the removed number :

[tex]let \: y = removed \: number[/tex]

[tex] \frac{35 - y}{4} = 6[/tex]

[tex]35 - y = 6 \times 4[/tex]

[tex]35 - y = 24[/tex]

[tex]35 - 24 = y[/tex]

[tex]y = 11[/tex]

Okay, let's slightly generalize this

Average of [tex]n[/tex] numbers is [tex]a[/tex]

and then [tex]r[/tex] numbers are removed, and you're asked to find the sum of these [tex]r[/tex] numbers.

Solution:

If average of [tex]n[/tex] numbers is [tex]a[/tex] then the sum of all these numbers is [tex]n\cdot a[/tex]

Now we remove [tex]r[/tex] numbers, so we're left with [tex](n-r)[/tex] numbers. and their. average will be [tex]{\text{sum of these } (n-r) \text{ numbers} \over (n-r)}[/tex] let's call this new average [tex] a^{\prime}[/tex]

For simplicity, say, sum of these [tex]r[/tex] numbers, which are removed is denoted by [tex]x[/tex] .

so the new average is [tex]\frac{\text{Sum of } n \text{ numbers} - x}{n-r}=a^{\prime}[/tex]

or, [tex] \frac{n\cdot a -x}{n-r}=a^{\prime}[/tex]

Simplify the equation, and solve for [tex]x[/tex] to get,

[tex] x= n\cdot a -a^{\prime}(n-r)=n(a-a^{\prime})+ra^{\prime}[/tex]

Hope you understand it :)

Question 2b only! Evaluate using the definition of the definite integral(that means using the limit of a Riemann sum

Answers

Answer:

Hello,

Step-by-step explanation:

We divide the interval [a b] in n equal parts.

[tex]\Delta x=\dfrac{b-a}{n} \\\\x_i=a+\Delta x *i \ for\ i=1\ to\ n\\\\y_i=x_i^2=(a+\Delta x *i)^2=a^2+(\Delta x *i)^2+2*a*\Delta x *i\\\\\\Area\ of\ i^{th} \ rectangle=R(x_i)=\Delta x * y_i\\[/tex]

[tex]\displaystyle \sum_{i=1}^{n} R(x_i)=\dfrac{b-a}{n}*\sum_{i=1}^{n}\ (a^2 +(\dfrac{b-a}{n})^2*i^2+2*a*\dfrac{b-a}{n}*i)\\[/tex]

[tex]=(b-a)^2*a^2+(\dfrac{b-a}{n})^3*\dfrac{n(n+1)(2n+1)}{6} +2*a*(\dfrac{b-a}{n})^2*\dfrac{n (n+1)} {2} \\\\\displaystyle \int\limits^a_b {x^2} \, dx = \lim_{n \to \infty} \sum_{i=1}^{n} R(x_i)\\\\=(b-a)*a^2+\dfrac{(b-a)^3 }{3} +a(b-a)^2\\\\=a^2b-a^3+\dfrac{1}{3} (b^3-3ab^2+3a^2b-a^3)+a^3+ab^2-2a^2b\\\\=\dfrac{b^3}{3}-ab^2+ab^2+a^2b+a^2b-2a^2b-\dfrac{a^3}{3} \\\\\\\boxed{\int\limits^a_b {x^2} \, dx =\dfrac{b^3}{3} -\dfrac{a^3}{3}}\\[/tex]

Other Questions
Es una _______ que t no puedas venir conmigo.lstimapena de muertepistoladelincuencia Big sleds must hold 3 children and small sleds must hold 2 children. If 17 children want to go sledding at the same time, how many of each type of sled is needed? If my primary DNA strand is ATACCGCAA a write the complimentary DNA strand A hypothesis test is to be performed in order to test the proportion of people in a population that have some characteristic of interest. Select all of the pieces of information that are needed in order to calculate the test statistic for the hypothesis test: A player has 15 hits in 34 times at bat and then getsanother hit. Did the batting average increase? Explain. Pls someone explain this to me Thank u. Which of the following activities meets the federal definitions of research? que es lo que ha hecho que en colombia el conflicto se haya multiplicado cada ves mas a medida que pasaban los aos help meh please I'm bad at math The altitude a(in feet) of a plane i minutes after liftoff is given bya = 34001 + 600. How many minutesafter liftoff is the planeat an altitude of21.000 feet? 17. Complete the following equation using , or =7 __ 24/2A. >B. Study the table below and answer the questions that followOutput units Total costsAverage costs Marginal costs(TC)(ATC)(MC)0100120201022814833411.364.389.545428.4464886756888729162.1Identify the value of the fixed costs.(12.2Give an example of a fixed cost.(12.32.4Briefly describe the term marginal cost.(2Explain the intersection point of ATC and MC using information in the table.( Sarah Meeham blends coffee for Tasti-Delight. She needs to prepare 170 pounds of blended coffee beans selling for $3.76 per pound. She plans to do this by blending together a high-quality bean costing $5.00 per pound and a cheaper bean at $2.00 per pound. To the nearest pound, find how much high-quality coffee bean and how much cheaper coffee bean she should blend What transportation mode has very high initial investment costs but gives a very low cost per mile for products that are highly specialized and require no packaging? Undulation in sandy material is calledcross-bedsripple marksmud cracksO fossils Qu compraste? El fin de semana fuiste a una venta de jardn. Escribe un prrafo para contar lo que hiciste en la venta. Usa los verbos de la lista en pretrito1. Yo __________ (creer) que siempre haba guantes de beisbol para todos en las tiendas de deportes.2.Yo __________ (pagar) con la tarjeta de crdito de mi madre y le voy a dar el dinero maana. 3. Yo no __________ (comprar) nada porque mi hermana menor usa su mano izquierda para echar la pelota y no vi ningn guante para alguien como ella. 4. Yo no __________ (regatear) porque no se permite regatear en esa tienda.5. Luego un dependiente de la tienda me habl. Me dijo que haba ms en Internet. Yo __________ (encontrar) un guante de beisbol muy bonito para mi hermana menor en el sitio web de la tienda.6. Despus de ir de compras, yo __________ (llegar) a casa contento.7. Este fin de semana, yo __________ (buscar) un guante de beisbol para mi hermana menor.8. Cuando no haba nada para ella, yo __________ (empezar) a llorar. if b 5/14, 7/10, 5/6, 11/15, 19/2 What does the photo reveal about how the war is going at this time? Do you believe the people in the photo are on the winning or losing side of the war at this time? Why do you believe this? Provide specific examples from the photo for support. Does your research about the photo support your conclusions about who is winning and how the war is going? Provide historical evidence and support for your analysis.If you cannot see the attachment, the picture is:"USS SHAW exploding during the Japanese raid on Pearl Harbor." December 7, 1941. Are you good at giving advice to people? Please give some suggestions for people in the following situations. Write down your suggestions using informal commands. You will use both affirmative and negative commands. Use a different verb for each command.Modelo: Cmo llevar una dieta saludable(S) Come muchas verduras (No) No bebas refrescos.Cmo sobrevivir la clase de espaolS_______________________[FILL]___________________________________________________[FILL]___________________________________________________[FILL]____________________________No_______________________[FILL]___________________________________________________[FILL]____________________________Cmo pasarlo bien en esta ciudadS_______________________[FILL]___________________________________________________[FILL]___________________________________________________[FILL]____________________________No_______________________[FILL]___________________________________________________[FILL]____________________________